You are on page 1of 11

PROBLEMAS SOBRE PONTOS

Davi Mximo (UFC) e Samuel Feitosa (UFC)


Nvel Avanado
Distribuir pontos num plano ou num espao uma tarefa que pode ser realizada de forma
muito arbitrria. Por isso, problemas sobre pontos podem ser de diversas naturezas. Nesse artigo,
trataremos as principais tcnicas para resolver esses tipos de problemas,
1. Fecho Convexo
Pense no seguinte: dados n pontos num plano, podemos escolher alguns deles formando o
nico polgono convexo que contm, junto com seu bordo e seu interior, todos os n pontos. Tal
afirmao pode ser provada por induo (que alias, uma ferramenta que sempre deve ser lembrada
em problemas de matemtica discreta em geral). Tal polgono chamado o fecho convexo desses n
pontos. Vamos ver que to pouco j nos ajuda bastante em alguns problemas sobre pontos.
PROBLEMA 1
Seja S um conjunto finito de pontos, no havendo trs colineares, tal que dados quaisquer 4 pontos
de S eles formam um quadriltero convexo. Mostre que S um conjunto de vrtices de um polgono
convexo.
SOLUO:
Seja H o fecho convexo de S.
H
B
C
.
P
A
Suponha um ponto P de S no interior de H. Escolha uma
triangulao de H (assim como o fecho convexo,
simples provar que todo polgono convexo pode ser
dividido por tringulos tendo como lados diagonais ou
lados do polgono, tente induo).
Assim, P fica no interior de algum tringulo ABC. Logo,
o quadriltero ABCP no convexo, absurdo!
Portanto, S no pode ter pontos no interior do seu fecho
convexo, donde S convexo, j que S no contm trs
pontos colineares.
Os prximos problemas so resolvidos similarmente.
PROBLEMA 2
Mostre que dados 5 pontos, no trs colineares, existe um quadriltero convexo com vrtices nesses
pontos.
PROBLEMA 3
Mostre que dado qualquer conjunto finito de pontos no plano existe uma reta por dois destes pontos
que divide o plano em dois semi-planos de modo que um desses semi-planos no contm nenhum
ponto do conjunto.
PROBLEMA 4
(Lista Cone Sul 2001) possvel que a reunio de um nmero finito de quadrilteros no convexos
seja um polgono convexo?
Podemos definir fecho convexo para um conjunto X qualquer do plano. Ele o menor conjunto
convexo que contm X.
Definio: O fecho convexo H de X a interseo de todos os conjuntos convexos do plano que
contm X.
Deixamos para o leitor a verificao dos seguintes fatos:
i-) H convexo
ii-) No caso de um conjunto com um nmero finito de pontos esta definio implica que H um
polgono convexo cujos vrtices pertencem a este conjunto.
PROBLEMA 5
Dado um conjunto de N discos de raios unitrios. Esses crculos podem se intersectar (mas no
coincidir). Mostre que existe um arco de comprimento maior ou igual a
N
2
pertencendo
circunferncia de um desses discos que no coberto por nenhum outro disco.
(IDIA DA SOLUO) Consideremos o fecho convexo H
desse conjunto de discos. Um arco que esteja na borda do fecho
convexo no pode ser coberto por outro disco. Mostre que a
juno de todos os arco no bordo de H um crculo de raio
unitrio. Como este crculo tem permetro 2 e no mximo
juntamos N arcos, pelo menos um dos arcos da juno maior
ou igual a
N
2
.
PROBLEMA 6
(OBM 96) Existe um conjunto A de n pontos ( 3 n ) em um plano tal que:
i) A no contm trs pontos colineares;
ii) dados quaisquer trs pontos pertencentes a A, o centro da circunferncia que contm estes pontos
tambm pertence a A?
Os prximos dois problemas so de IMO e podem ser resolvidos usando s fecho convexo (na
realidade, muita raa tambm, que algo imprescindvel em qualquer problema, principalmente de
IMO).
PROBLEMA 7
(IMO 99/1) Determine todos os conjuntos finitos S de pontos do plano com pelo menos trs
elementos que satisfazem a seguinte condio:
Para quaisquer dois pontos distintos A e B de S, a mediatriz do segmento AB um eixo de simetria
de S.
(Veja a soluo desse problema por Fabrcio Siqueira Benevides na Eureka! N.6)
PROBLEMA 8
(IMO 95/3) Determine todos os inteiros 3 > n para os quais existem n pontos
n
A A A , , ,
2 1
no
plano, e nmeros reais
n
r r r , ,
2 1
satisfazendo as condies:
no h trs pontos
k j i
A A A , ,
colineares;
para cada tripla i ,j, k ( n k j i < < 1 ) o tringulo
k j i
A A A
tem rea
k j i
r r r + +
.
SOLUO:
Vamos fazer o caso 5 n . Considere, dentre os n pontos, cinco pontos
5 4 3 2 1
, , , , A A A A A e seu
fecho convexo . Temos trs casos:
1 Caso: O Fecho Convexo um tringulo.
Podemos assumir que tal fecho o tringulo
3 2 1
A A A ,
4
A e
5
A esto no interior de
3 2 1
A A A , com
5
A fora de
4 2 1
A A A e
4
A fora de
4 2 1
A A A (faa uma figura). Podemos supor que os tringulos
4 2 1
A A A e
2 3 5
A A A tm interiores disjuntos. Seguindo nossa notao para reas, temos:
1 2 3 1 2 3 1 2 4 2 3 5 1 2 4 2 3 5
[ ] [ ] [ ] , r r r AA A AA A A A A r r r r r r + + > + + + + + + donde
4 2 5 2 4 5
0 [ ], r r r A A A > + + absurdo!
2 Caso: O Fecho Convexo um quadriltero
Suponha
5
A no interior do fecho convexo
4 3 2 1
A A A A . Note que
] [ ] [ ] [ ] [ ] [
4 3 2 4 2 1 4 3 1 3 2 1 4 3 2 1
A A A A A A A A A A A A A A A A + +
e portanto,
4 2 3 1 3 4 2 4 2 1 1 4 3 3 2 1
) ( ) ( ) ( ) ( r r r r r r r r r r r r r r r r + + + + + + + + + + + + .
Logo, ) ( 3 ] [ 2
4 3 2 1 4 3 2 1
r r r r A A A A + + + . Tambm,
] [ ] [ ] [ ] [ ] [
5 1 4 5 4 3 5 3 2 5 2 1 4 3 2 1
A A A A A A A A A A A A A A A A + + +
Logo, temos 0 12 ] [ 8 ) (
4 3 2 1 4 3 2 1 5
< + + + A A A A r r r r r . Agora, observe que como
5 3 1
, , A A A no so colineares, podemos supor um dos lados de < 180
5 3 1
A A A . Ento, um dos
quadrilteros
4 3 5 1
A A A A ,
2 3 5 1
A A A A convexo. Digamos,
4 3 5 1
A A A A convexo. Ento, temos
5 4 3 1
r r r r + + e portanto, ficamos com
5 2
r r . Analogamente, usando que
5 4 2
, , A A A no so
colineares, temos
1 5
r r ou
3
r . Assim, trs dos nmeros
5 4 , 3 2 1
, , , r r r r r
so negativos, obtendo uma
rea negativa. Absurdo!
3 Caso: O Fecho convexo um pentgono
Suponha que
1
r seja o menor deles. Traando uma paralela l por
1
A reta
4 3
A A . Como
] [ ] [
4 3 2 4 3 2 4 3 1 4 3 1
A A A r r r r r r A A A + + + + ,
2
A pertence a l ou ao semiplano definido por
l oposto ao
4 3
A A e, analogamente
5
A . Como
5 2 1
, , A A A no podem estar todos em l , temos
> 180
5 1 2
A A A , absurdo! .
Logo, 4 n . Um exemplo para 4 n um quadrado
4 3 2 1
A A A A de lado 1 com
6 1
4 3 2 1
r r r r .
Finalizamos essa parte com dois problemas bonitinhos.
PROBLEMA 9
(USAMO 2005) Seja n um inteiro positivo maior que 1. Suponha que so dados 2n pontos no plano,
no havendo trs colineares. Suponha que n dos 2n so pintados de azul e os outros n de vermelho.
Uma reta no plano dita balanceada se passa por um ponto azul e um ponto vermelho, e o nmero
de pontos azuis em cada um de seus lados igual ao nmero de pontos vermelhos. Prove que
existem pelo menos duas retas balanceadas.
DICA: Prove que cada ponto do fecho convexo dos pontos est em pelo menos uma reta
balanceada.
PROBLEMA 10
(Kmal 2002) Dado um conjunto qualquer de pontos no plano, no contendo trs colineares, prove
que possvel colorir os pontos com duas cores (azul e vermelho) tal que todo semiplano contendo
pelo menos trs pontos do conjunto contenha pelo menos um ponto de cada cor.
2. Princpio das Casas dos Pombos
O princpio da casas dos Pombos, PCP, importante e deve ser lembrado sempre. Ele usado para
provar existncias (...se 1 + n pombos esto em n casas, existe pelo menos uma casa contendo pelo
menos dois pombos...). Nossos dois primeiros problemas dessa sesso so apenas verses
dificultadas daquele exerccio clssico do PCP. : dados cincos pontos num quadrado unitrio,
existem dois cuja distncia entre eles menor que
2
2
.
PROBLEMA 11
(Japo 97) Prove que entre quaisquer dez pontos no interior de um crculo de dimetro 5, existem
dois cuja distncia entre eles menor que 2.
PROBLEMA 12
(Coria 97) Prove que entre quaisquer quatro pontos no interior de um crculo unitrio, existem dois
deles cuja distncia menor que 2 .
PROBLEMA 13
(Rioplatense 2002) Daniel escolhe um inteiro positivo n e diz a Ana. Com esta informao, Ana
escolhe um inteiro k e diz a Daniel. Daniel traa ento n circunferncias em um papel e escolhe k
pontos distintos com a condio de que cada um deles pertena a alguma das circunferncias que
traou. Em seguida, apaga as circunferncias que traou, sobrando visveis apenas os k pontos que
marcou. A partir desses pontos, Ana deve reconstruir pelo menos uma das circunferncias que
Daniel traou. Determinar qual o menor valor de k que permite Ana alcanar seu objetivo
independente de como Daniel escolha as n circunferncias e os k pontos.
SOLUO:
O valor mnimo
1 2
2
+ n k
.
1 Passo:
1 2
2
+ n k
suficiente.
Se so dados
1 2
2
+ n
pontos marcados por Daniel, como estes pontos so distribudos em n
circunferncias, pelo Princpio das Casas dos Pombos, pelo menos 2n+1 deles esto em uma mesma
circunferncia traada por Daniel. Ento, se Ana traa todas as circunferncias determinadas por
estes
1 2
2
+ n
, haver uma delas, digamos , com pelo menos 2n + 1 dos pontos. Como estes 2n +
1 pontos provm das n circunferncias de Daniel, trs deles esto numa mesma circunferncia
traada por ele, digamos . Logo, e tm pelo menos trs pontos em comum, e portanto, so
a mesma circunferncia (abusando da notao: ). Assim, Ana consegue determinar umas das
circunferncias traadas por Daniel.
2 Passo: Se
1 2
2
+ < n k
, Daniel pode traar circunferncias e escolher k pontos de modo a tornar
impossvel para Ana determinar tais circunferncias.
Basta considerar
2
2n k
:
1

Traamos n circunferncias concntricas


n
, , ,
2 1
e
outras n circunferncias
n
, , ,
2 1
duas a duas
disjuntas, de modo que
i
corta
j

em dois pontos
distintos, para i, j = 1,2,...,n. H exatamente
2
2n
pontos
de interseco. Se Daniel marca estes pontos e apagas
suas circunferncias, Ana no conseguir reconstruir com
certeza nenhuma das circunferncias, pois Daniel pode ter
traado inicialmente tanto
n
, , ,
2 1
quanto
n
, , ,
2 1
.
PROBLEMA 14
(Rioplatense 1999) Dois jogadores A e B disputam o seguinte jogo: A escolhe um ponto de
coordenadas inteiras do plano e o pinta de verde; em seguida B escolhe 10 pontos de coordenadas
inteiras, ainda no coloridos e os pinta de amarelo. O jogo continua assim com as mesmas regras: A
e B escolhem um e dez pontos ainda no coloridos e os pintam de verde e amarelo, respectivamente.
(a) O objetivo de A obter
2
111
pontos verdes que sejam as intersees de 111 retas horizontais e
111 retas verticais (i.e., paralelas aos eixos de coordenadas). O objetivo de B impedir-lhe.
Determine qual dos jogadores tem uma estratgia vencedora que lhe assegura seu objetivo.
(b) O objetivo de A obter quatro pontos verdes que sejam vrtices de um quadrado de lados
paralelos aos eixos coordenados. O objetivo de B impedir-lhe. Determine qual dos jogadores tem
uma estratgia vencedora que lhe assegura seu objetivo.
3. Idias Extremais
Na matemtica em geral, problemas de existncia so muito comuns e importantes. So aqueles
problemas que nos pedem para provar que a existncia de alguma coisa. Na seo anterior, no
explicitamente, nos deparamos com problemas desse tipo. E no foi para vender o artigo que
iniciamos ambas as sees falando da importncia dessas idias, mas pelo o fato de que o PCP e o
Princpio Extremal juntos so as ferramentas mais indispensveis para o ataque desses problemas.
Mas afinal, que Princpio Extremal esse? Digamos que temos um problema onde nos pedido
para provar a existncia de um elemento satisfazendo uma certa propriedade P. Ento, ns
escolhemos um elemento que satisfaz maximalmente ou minimalmente, ou seja, extremalmente
(ser que acabamos de inventar essas palavras?) uma outra propriedade Q, no acidentalmente
ligada com a desejada propriedade P. O que ser que isso nos d? Vejamos alguns problemas.
PROBLEMA 15
(Austrlia 91) So dados 3 n pontos no plano tais que a rea de um tringulo formado por
quaisquer trs deles no mximo 1. Prove que os
n
pontos esto em um tringulo de rea no
mximo 4.
SOLUO:
Sejam
n
P P P , , ,
2 1
os n pontos. Dentre os tringulos considerados, seja ABC o de maior rea (o
cara com a propriedade Q). Considere por A uma reta a paralela a BC. Sendo assim, qualquer outro
ponto
i
P
A B
C
Z
Y X
c
b
deve estar no mesmo semiplano de B e C definido por A,
pois do contrrio teramos um absurdo
] [ ] [ ABC PBC >

(aqui [X], denota a rea de X) . Analogamente,
considerando as retas b e c por B e C paralelas a AC e AB,
respectivamente, conclumos que todos os pontos P
devem estar no tringulo XYZ (acompanhe a figura ao
lado). Como,
4 1 4 ] [ 4 ] [ < ABC XYZ
, o resultado segue. (isto ,
XYZ satisfaz a propriedade P).
PROBLEMA 16
(Putnam 1979) Sejam n 2 pontos no plano escolhidos de modo que quaisquer 3 no so colineares,
n deles so pintados de vermelho e n deles so pintados de azul. Prove que possvel parear os
pontos usando segmentos ligando cada ponto vermelho a exatamente um ponto azul de modo que
esses segmentos no se cortem.
SOLUO:
Existem
2
n
maneiras de parear esses pontos. claro que alguns desses pareamentos no cumprem
a condio do enunciado. Olhemos em cada pareamento a soma dos seus segmentos. Escolha o
pareamento que tem soma mnima. Suponha que nele existem dois segmentos AB e CD que se
cortam (com A e C vermelhos)
C
B
D
A
O
Pela desigualdade triangular temos:
CB AD CD AB
CB OC OB
CD OD AO
+ > +

> +
> +
Logo se trocarmos AB e CD por AD e CB
diminuiremos nossa soma. Assim neste pareamento no
temos dois segmentos que se cortam.
PROBLEMA 17
(Teorema de Sylvester) Um conjunto S de pontos no plano tem a seguinte propriedade: qualquer
reta passando por 2 pontos passa tambm por um terceiro. Mostre que todos os pontos esto sobre
uma reta.
SOLUO:
Considere o conjunto L das retas que passam por dois pontos de S. Cada ponto de S tem uma
distncia associada a cada reta de L. Como L e S so conjuntos finitos ento temos um nmero
finito distncias. Considere o par ) , ( s l do ponto L l S s e com a menor distncia no nula
associada. Como l passa por dois pontos de S ento dever passar por um terceiro. Pelo menos dois
pontos de S , digamos A e B, devero estar em um mesmo lado de l determinado por P (p da
perpendicular de
s
at l ).
s
l
P
A
B
m
Suponhamos que A esteja entre B e P. Seja m a reta que
passa por B e
s
ento:
distncia ( , ) distncia ( , ) A m P m
<
distncia ( , ) s l <
Absurdo!
Assim todas as distncias associadas tm que ser zero! Todos os pontos so colineares!
A seguir, veja como usar o Teorema de Sylvester.
PROBLEMA 18
So dados ) 3 ( N pontos no plano, nem todos colineares. Mostre que so necessrios pelo menos
n retas para unir todos os possveis pares de pontos.
SOLUO:
Vamos tentar usar induo. Se 3 N os trs pontos formaro um tringulo. As retas suportes dos
trs lados desse tringulo satisfazem nossa afirmao. Suponha que a afirmao seja vlida para
k N . Considere um conjunto T de 1 + k N pontos. Como nem todos esses pontos esto sobre
uma mesma reta decorre do teorema de Sylvester que existe uma reta que passa por apenas dois
pontos (A e B) do conjunto. Pelo menos um dos conjuntos } { \ ou } { \ B T A T no poder ter todos
os seus k pontos colineares. Ento pela hiptese teremos pelo menos k retas, mas a reta AB no
foi contada, assim a afirmao tambm verdadeira para 1 + k N .
PROBLEMA 19
Dado um conjunto finito S de pontos no plano onde no existem quatro sobre um mesmo crculo e
nem todos esto sobre uma mesma reta. Mostre que existe um crculo que passa por trs desses
pontos e no contm nenhum ponto de S em seu interior.
PROBLEMA 20
(Ibero 93) Prove que para qualquer polgono convexo de rea 1, existe um paralelogramo de rea
dois que o contm.

PROBLEMA 21
(OBM 94) Considere todos os crculos cujas circunferncias passam por trs vrtices consecutivos
de um polgono convexo. Prove que um destes crculos contm todo o polgono.
PROBLEMA 22
(Rioplatense 97) Agustina e Santiago jogam o seguinte jogo sobre uma folha retangular:
Agustina diz um nmero n. Santiago, ento marca n pontos sobre a folha . Em seguida, Agustina
escolhe alguns dos pontos marcados por Santiago. Santiago ganha o jogo se consegue desenhar um
retngulo com lados paralelos aos da folha, que contenha todos os escolhidos por Agustina e
nenhum dos restantes. Do contrrio, Agustina ganha.
Qual o menor nmero que deve escolher Agustina para assegurar-se da vitria, independente como
jogue Santiago?
PROBLEMA 23
(Rssia 2000) So dados 2n + 1 segmentos em uma linha reta. Cada segmento intersecta pelo
menos n outros. Prove que um desses segmentos intersecta todos os outros.
PROBLEMA 24
(Japo 2002) dado um conjunto S de 2002 pontos no plano xy, no havendo dois deles com a
mesma abscissa x ou ordenada y. Para quaisquer dois desses pontos P e Q, considere o retngulo
cuja diagonal PQ e cujos lados so paralelos aos eixos. Denotemos por
PQ
W
o nmero de pontos
de S no interior desse retngulo, sem contar com P e Q. Determine o maior valor N possvel que
satisfaz: no importa como os pontos de S esto arranjados, existe pelo menos um par P e Q deles
com
N W
PQ

.
PROBLEMA 25
Dados 2n + 2 pontos no plano, no havendo trs colineares, prove que existem dois deles que
determinam uma reta que, dos 2n pontos restantes, separa n em um semi-plano e os outros n no
outro semi-plano.
PROBLEMA 26
(Banco IMO 93) Dados 2n + 3 pontos num plano, no havendo trs colineares nem quatro
concclicos, prove que podemos escolher trs deles de modo o crculo passando por estes tem n dos
pontos restantes no seu interior e n no exterior.
SOLUO: Basta considerar a figura abaixo. Deixamos os detalhes para o leitor.
1
A
1 + n
A
1 2 + n
A
2 2 n
A P
+

2 3 n
A Q
+

PROBLEMA 27
So dados n pontos num plano. Em cada ponto mdio de um segmento ligando dois desses pontos,
colocamos um marcador. Prove que pelo menos 3 2 n marcadores so utilizados.
4. Problemas de Coberturas
Nos problemas sobre pontos at agora, ficou claro que um pouco de geometria (sinttica, analtica,
trigonomtrica ou utilizando o plano complexo) pode ser til. Finalizaremos esse artigo com uma
seo falando um pouco disso, em particular, fazendo coberturas com crculos.
PROBLEMA 28
Seja C um crculo de raio 16 e A um anel tendo raio interno 2 e raio externo 3. Agora suponha que
um conjunto S de 650 pontos selecionado dentro de C. Prove que, no importa como os pontos de
S so selecionados dentro de C, o anel A pode ser colocado de modo a cobrir pelo menos 10 pontos
de S.
SOLUO:
Queremos mostrar que existe um ponto X no plano que possui uma distncia maior que 2 e menor
que 3 pelo menos 10 pontos de S. Sobre cada ponto de S coloque um anel A. Basta mostrarmos
que existe um ponto X que est no interior de pelo menos 10 desses anis. As intersees desses
anis produzem pequenas regies (veja a figura).
Veja que existem trs pequenas regies que esto em dois
anis e uma que est em trs. Somando as reas dos trs
anis contaremos trs regies duas vezes e uma trs
vezes. Somando a rea de cada anel temos
3250 ) 4 9 .( 650 .Aumentando o raio do crculo
C para 19 poderemos cobrir todos esses anis. Se cada
pequena regio foi contada no mximo 9 vezes
contaremos no mximo 9 vezes a rea desse novo
crculo , ou seja,
3250 3249 19 . 9
2
<
.
Assim existir uma pequena regio contida em pelo menos 10 anis. Basta escolhermos um ponto X
dessa regio.
PROBLEMA 29
(Teste de Seleo da Romnia para a IMO 1978) M um conjunto de n 3 pontos no plano tal que
a maior distncia entre quaisquer dois desses pontos 1 unidade. Prove que:
a. Para quaisquer 4 pontos de M, a distncia entre algum par de pontos pelo menos
2
1
b. Algum crculo de raio
2
3

cobre todo o conjunto M.


c. Existe algum par entre os n 3 pontos de M cuja distncia entre eles no mximo
) 3 3 (
4
n
SOLUO:
a. Vamos tentar arranjar um tringulo no acutngulo em M. Considere o fecho convexo de quatro
pontos de M. Podemos ter um quadriltero (degenerado quando trs pontos forem colineares) ou um
tringulo com um ponto de M em seu interior. No caso do quadriltero como pelo menos um dos
quatro ngulos internos

90
basta escolhermos o vrtice com este ngulo e os adjacentes a ele.
No caso do tringulo vamos olhar para o ponto no interior. Esse ponto olha para os trs lados do
tringulo com ngulos que somados resultam em

360
. Pelo menos um deles

120
. Seja XYZ
um tringulo com

90 XYZ
. Pela lei dos cossenos temos:
2 2 2 2 2
cos 2 z x XYZ xz z x y + + . Como
2
1
z ou 1 x y
.
b. Seja AB r a maior distncia entre dois pontos de M. Tracemos crculos de raio r centrados
em A e B. M dever estar contido em cada um desse crculos. Ento M dever estar contido na
regio de interseo entre eles. Tracemos um crculo de raio
3 3
2 2
r
centrado no ponto mdio C
de AB. Veja que este novo crculo cobre a regio de interseo.
Y
M
A C
B
X
c. Vamos usar a mesma idia do problema dos anis. Se dois
ponto de M esto em um crculo de raio r ento a distncia
entre eles no pode ser maior ou igual a r 2 . Ento nosso
objetivo mostrar que existem dois pontos de M dentro de um
crculo de raio
) 3 3 (
2
n

Seja C um crculo de raio
2
3
que cobre M. Centrado em cada ponto de M tracemos um crculo de
raio r . Suponha que Z um ponto na interseo de dois desses crculos. Ento o crculo de centro Z
e raio r cobre dois pontos de M(os centros dos crculos que cobriam Z). Como mostrar pelo menos
dois desses crculos que traamos iro se intersectar para
) 3 3 (
2

n
r
? Vamos aumentar o raio
de C e obter um crculo D de raio r +
2
3
com mesmo centro. Todos esses crculos estaro contidos
em D. Se a rea de D for menor que a soma das reas de cada crculo com certeza pelo menos dois
deles tero interseo. Mas isso acontece se
2
2
3
3 .
2
n r r
_
< +


,
Agora basta fazermos o estudo
do sinal. Como a maior raiz 0 1 3 e
) 1 3 ( 2
3 3
>

+
n
n
n
se
3 3 3
OK!
2(3 1) 2(3 3)
n
r r
n n
+
> >

.
(Veja que podemos melhorar um pouco a cota do problema, pois
2
3
2 > ).
PROBLEMA 30
(Ibero 97) Seja } ,..., , {
1997 2 1
P P P P um conjunto de 1997 pontos no interior de um crculo de raio
1, com
1
P sendo o centro do crculo. Para 1997 ,..., 2 , 1 k , seja
k
x a distncia de
k
P ao ponto de
P mais prximo de
k
P . Mostre que:
9 ...
2
1997
2
2
2
1
+ + + x x x .
SOLUO:
P
1
P
n
Note que 1
k
x , para todo k.
Para cada ponto
k
P , considere uma circunferncia
k
de
centro
k
P e raio
2
k
x
.
Sendo assim, todos essas circunferncias se tocam em no
mximo 1 ponto e esto no interior de uma circunferncia de
centro
1
P e raio 3/2. Logo, ( )
2
2 1
2
3
] [ ] [ ] [ + + +
k

donde:
( )
2
2 2
2
2
1
2
3
2 2 2

,
_

+ +

,
_

,
_

n
x x x
9
2 2
2
2
1
+ + +
n
x x x .
PROBLEMA 31
(IMO 89) So dados n e k inteiros positivos e um conjunto S de n pontos no plano tais que
(i) no h trs pontos em S colineares,
(ii) Para qualquer ponto P de S existem pelo menos k pontos de S eqidistantes de P.
Prove que n k 2
2
1
+ < .
PROBLEMA 32
(Ibero 98) Encontre o maior inteiro
n
para o qual existem pontos
n
P P P ,..., ,
2 1
no plano e nmeros
reais
n
r r r ,..., ,
2 1
tais que a distncia entre
i
P e
j
P

j i
r r +
.
Referncias Bibliogrficas:
[1] Revista Eureka! N6.
[2] Ross Honsberger, Mathematical Gems Vol.I, The Dolciani Mathematical Expositions, MAA.
[3] Andreescu, Feng, Mathematical Olympiads: Olympiad problems from around the world, 1999-2000,
MAA 2000.
[4] Marcin Kuczma , International Mathematical Olympiads, 1986-1999, MAA 2003.
[5] www.mathlinks.ro

You might also like